Showing two linear functionals are the same up to constant “$g(A) = qtr(A)$”












1












$begingroup$


The original problem is this




If $g: M_2(mathbb{R}) to mathbb{R}$ satisfies $g(AB) = g(BA)$, then there is a constant $k$ such that $g(A) = ktr(A)$ where $tr$ is the trace.




So after reading a bit, this identification is given when messing a bit with their kernels



Is there a more obvious solution? It wasn't clear to me to look at this general case.










share|cite|improve this question









$endgroup$












  • $begingroup$
    I am just curious. Can you really show directly that the kernel of $g$ is the same as the kernel of $A$?
    $endgroup$
    – user9077
    Dec 13 '18 at 0:14










  • $begingroup$
    You mean the trace? G doesn’t have an explicit map, so I can’t imagine how
    $endgroup$
    – Hawk
    Dec 13 '18 at 4:59










  • $begingroup$
    I mean if we want to use the result from the link we need to show that g and tr have the same kernel right? So I thought you know how to show that the kernel of g is the set of all matrix of zero trace.
    $endgroup$
    – user9077
    Dec 13 '18 at 7:29
















1












$begingroup$


The original problem is this




If $g: M_2(mathbb{R}) to mathbb{R}$ satisfies $g(AB) = g(BA)$, then there is a constant $k$ such that $g(A) = ktr(A)$ where $tr$ is the trace.




So after reading a bit, this identification is given when messing a bit with their kernels



Is there a more obvious solution? It wasn't clear to me to look at this general case.










share|cite|improve this question









$endgroup$












  • $begingroup$
    I am just curious. Can you really show directly that the kernel of $g$ is the same as the kernel of $A$?
    $endgroup$
    – user9077
    Dec 13 '18 at 0:14










  • $begingroup$
    You mean the trace? G doesn’t have an explicit map, so I can’t imagine how
    $endgroup$
    – Hawk
    Dec 13 '18 at 4:59










  • $begingroup$
    I mean if we want to use the result from the link we need to show that g and tr have the same kernel right? So I thought you know how to show that the kernel of g is the set of all matrix of zero trace.
    $endgroup$
    – user9077
    Dec 13 '18 at 7:29














1












1








1





$begingroup$


The original problem is this




If $g: M_2(mathbb{R}) to mathbb{R}$ satisfies $g(AB) = g(BA)$, then there is a constant $k$ such that $g(A) = ktr(A)$ where $tr$ is the trace.




So after reading a bit, this identification is given when messing a bit with their kernels



Is there a more obvious solution? It wasn't clear to me to look at this general case.










share|cite|improve this question









$endgroup$




The original problem is this




If $g: M_2(mathbb{R}) to mathbb{R}$ satisfies $g(AB) = g(BA)$, then there is a constant $k$ such that $g(A) = ktr(A)$ where $tr$ is the trace.




So after reading a bit, this identification is given when messing a bit with their kernels



Is there a more obvious solution? It wasn't clear to me to look at this general case.







linear-algebra






share|cite|improve this question













share|cite|improve this question











share|cite|improve this question




share|cite|improve this question










asked Dec 12 '18 at 0:28









HawkHawk

5,5151139104




5,5151139104












  • $begingroup$
    I am just curious. Can you really show directly that the kernel of $g$ is the same as the kernel of $A$?
    $endgroup$
    – user9077
    Dec 13 '18 at 0:14










  • $begingroup$
    You mean the trace? G doesn’t have an explicit map, so I can’t imagine how
    $endgroup$
    – Hawk
    Dec 13 '18 at 4:59










  • $begingroup$
    I mean if we want to use the result from the link we need to show that g and tr have the same kernel right? So I thought you know how to show that the kernel of g is the set of all matrix of zero trace.
    $endgroup$
    – user9077
    Dec 13 '18 at 7:29


















  • $begingroup$
    I am just curious. Can you really show directly that the kernel of $g$ is the same as the kernel of $A$?
    $endgroup$
    – user9077
    Dec 13 '18 at 0:14










  • $begingroup$
    You mean the trace? G doesn’t have an explicit map, so I can’t imagine how
    $endgroup$
    – Hawk
    Dec 13 '18 at 4:59










  • $begingroup$
    I mean if we want to use the result from the link we need to show that g and tr have the same kernel right? So I thought you know how to show that the kernel of g is the set of all matrix of zero trace.
    $endgroup$
    – user9077
    Dec 13 '18 at 7:29
















$begingroup$
I am just curious. Can you really show directly that the kernel of $g$ is the same as the kernel of $A$?
$endgroup$
– user9077
Dec 13 '18 at 0:14




$begingroup$
I am just curious. Can you really show directly that the kernel of $g$ is the same as the kernel of $A$?
$endgroup$
– user9077
Dec 13 '18 at 0:14












$begingroup$
You mean the trace? G doesn’t have an explicit map, so I can’t imagine how
$endgroup$
– Hawk
Dec 13 '18 at 4:59




$begingroup$
You mean the trace? G doesn’t have an explicit map, so I can’t imagine how
$endgroup$
– Hawk
Dec 13 '18 at 4:59












$begingroup$
I mean if we want to use the result from the link we need to show that g and tr have the same kernel right? So I thought you know how to show that the kernel of g is the set of all matrix of zero trace.
$endgroup$
– user9077
Dec 13 '18 at 7:29




$begingroup$
I mean if we want to use the result from the link we need to show that g and tr have the same kernel right? So I thought you know how to show that the kernel of g is the set of all matrix of zero trace.
$endgroup$
– user9077
Dec 13 '18 at 7:29










2 Answers
2






active

oldest

votes


















1












$begingroup$

Let $E_1=begin{pmatrix}1&0\0&0end{pmatrix},E_2=begin{pmatrix}0&1\0&0end{pmatrix},E_3=begin{pmatrix}0&0\1&0end{pmatrix}$ and $E_4=begin{pmatrix}0&0\0&1end{pmatrix}$.



If we can show that $$g(E_1)=g(E_4)=k text{ and } g(E_2)=g(E_3)=0,quad (*)$$ then
$$gbegin{pmatrix}a&b\c&dend{pmatrix}=ag(E_1)+bg(E_2)+cg(E_3)+dg(E_4)=k(a+d)=kcdottext{tr}(A)$$
as we want. Hence it is enough to show that $g$ satisfies $(*)$.



Note that by the condition $g(AB)=g(BA)$ we have
$$g(E_2)=g(E_1E_2)=g(E_2E_1)=g(0)=0$$
and
$$g(E_3)=g(E_4E_3)=g(E_3E_4)=g(0)=0 $$
Now
$(E_1+E_3)(E_1+E_2)=E_1+E_2+E_3+E_4$ and $(E_1+E_2)(E_1+E_3)=2E_1$. Then
$$g(2E_1)=g(E_1+E_2+E_3+E_4)=g(E_1)+g(E_4)$$ which implies that
$$g(E_1)=g(E_4)=k text{ for some } kin mathbb{R}.$$
Hence we are done.






share|cite|improve this answer









$endgroup$













  • $begingroup$
    So I just checked again and apparently the original problem was stated for $M_n$, I just wrote it $2$ because that's what I wrote on my paper when solving the question and translate it incorrectly.
    $endgroup$
    – Hawk
    Dec 12 '18 at 11:50










  • $begingroup$
    Oh no. Maybe we can use the same idea?
    $endgroup$
    – user9077
    Dec 12 '18 at 12:04










  • $begingroup$
    Well i ruled that out since the columns and rows can get large. For $n =4$ you have to do 16 cases...for case $n$ it is general to talk about, but I found this problem in an exam paper...so something tells me our method here won't be applicable.
    $endgroup$
    – Hawk
    Dec 12 '18 at 12:25










  • $begingroup$
    Well, it can be done, I just tried it. We just need to choose $A,B$ wisely when using $g(AB)=g(BA)$.
    $endgroup$
    – user9077
    Dec 12 '18 at 12:53



















0












$begingroup$

We can use the same idea if $g:M_n(mathbb{R})tomathbb{R}$. The idea is that if $B$ is a basis of $M_n$, also $g$ and $text{tr}$ agree on $B$ up to some constant multiple $alpha$, then $$g(A)=alphacdot text{tr}(A) $$ for all $Ain M_n$.



Let $E_{i,j}$ be the matrix where all the entries are zero except it is 1 at the position $(i,j)$. Let $T_k=text{diag}(underbrace{1,1,ldots,1}_k,0,ldots,0)$. Notice that for $ineq j$ we have $E_{i,j}T_j=E_{i,j}$ while $T_jE_{i,j}=0$. Therefore for all $ineq j$ we have $g(E_{i,j})=0$.



Now let $J_k$ be a $ktimes n$ matrix where all the entries are 1. Define an $ntimes n$ matrix $S_k$ as a block matrix $$S_k:=begin{pmatrix}J_k\0end{pmatrix}$$.



Note that since $g(E_{i,j})=0$ for $ineq j$, we have $g(S_k)=g(T_k)$ for all $k$.



Now
begin{align*}
S_kS_1&=S_k \ S_1S_k&=kS_1
end{align*}



Hence $g(T_k)=g(S_k)=g(kS_1)=kg(S_1)=kg(T_1)$. Notice that the trace map also satisfies $text{tr}(T_k)=k$. By taking $alpha=g(T_1)$ then $g$ and $alpha cdot text{tr}$ are two linear maps that agree on $B={E_{i,j}mid ineq j}cup {T_kmid k=1,..,n}$ which is a basis of $M_n$. Hence $g=alphacdot text{tr}.$






share|cite|improve this answer









$endgroup$













    Your Answer





    StackExchange.ifUsing("editor", function () {
    return StackExchange.using("mathjaxEditing", function () {
    StackExchange.MarkdownEditor.creationCallbacks.add(function (editor, postfix) {
    StackExchange.mathjaxEditing.prepareWmdForMathJax(editor, postfix, [["$", "$"], ["\\(","\\)"]]);
    });
    });
    }, "mathjax-editing");

    StackExchange.ready(function() {
    var channelOptions = {
    tags: "".split(" "),
    id: "69"
    };
    initTagRenderer("".split(" "), "".split(" "), channelOptions);

    StackExchange.using("externalEditor", function() {
    // Have to fire editor after snippets, if snippets enabled
    if (StackExchange.settings.snippets.snippetsEnabled) {
    StackExchange.using("snippets", function() {
    createEditor();
    });
    }
    else {
    createEditor();
    }
    });

    function createEditor() {
    StackExchange.prepareEditor({
    heartbeatType: 'answer',
    autoActivateHeartbeat: false,
    convertImagesToLinks: true,
    noModals: true,
    showLowRepImageUploadWarning: true,
    reputationToPostImages: 10,
    bindNavPrevention: true,
    postfix: "",
    imageUploader: {
    brandingHtml: "Powered by u003ca class="icon-imgur-white" href="https://imgur.com/"u003eu003c/au003e",
    contentPolicyHtml: "User contributions licensed under u003ca href="https://creativecommons.org/licenses/by-sa/3.0/"u003ecc by-sa 3.0 with attribution requiredu003c/au003e u003ca href="https://stackoverflow.com/legal/content-policy"u003e(content policy)u003c/au003e",
    allowUrls: true
    },
    noCode: true, onDemand: true,
    discardSelector: ".discard-answer"
    ,immediatelyShowMarkdownHelp:true
    });


    }
    });














    draft saved

    draft discarded


















    StackExchange.ready(
    function () {
    StackExchange.openid.initPostLogin('.new-post-login', 'https%3a%2f%2fmath.stackexchange.com%2fquestions%2f3036061%2fshowing-two-linear-functionals-are-the-same-up-to-constant-ga-qtra%23new-answer', 'question_page');
    }
    );

    Post as a guest















    Required, but never shown

























    2 Answers
    2






    active

    oldest

    votes








    2 Answers
    2






    active

    oldest

    votes









    active

    oldest

    votes






    active

    oldest

    votes









    1












    $begingroup$

    Let $E_1=begin{pmatrix}1&0\0&0end{pmatrix},E_2=begin{pmatrix}0&1\0&0end{pmatrix},E_3=begin{pmatrix}0&0\1&0end{pmatrix}$ and $E_4=begin{pmatrix}0&0\0&1end{pmatrix}$.



    If we can show that $$g(E_1)=g(E_4)=k text{ and } g(E_2)=g(E_3)=0,quad (*)$$ then
    $$gbegin{pmatrix}a&b\c&dend{pmatrix}=ag(E_1)+bg(E_2)+cg(E_3)+dg(E_4)=k(a+d)=kcdottext{tr}(A)$$
    as we want. Hence it is enough to show that $g$ satisfies $(*)$.



    Note that by the condition $g(AB)=g(BA)$ we have
    $$g(E_2)=g(E_1E_2)=g(E_2E_1)=g(0)=0$$
    and
    $$g(E_3)=g(E_4E_3)=g(E_3E_4)=g(0)=0 $$
    Now
    $(E_1+E_3)(E_1+E_2)=E_1+E_2+E_3+E_4$ and $(E_1+E_2)(E_1+E_3)=2E_1$. Then
    $$g(2E_1)=g(E_1+E_2+E_3+E_4)=g(E_1)+g(E_4)$$ which implies that
    $$g(E_1)=g(E_4)=k text{ for some } kin mathbb{R}.$$
    Hence we are done.






    share|cite|improve this answer









    $endgroup$













    • $begingroup$
      So I just checked again and apparently the original problem was stated for $M_n$, I just wrote it $2$ because that's what I wrote on my paper when solving the question and translate it incorrectly.
      $endgroup$
      – Hawk
      Dec 12 '18 at 11:50










    • $begingroup$
      Oh no. Maybe we can use the same idea?
      $endgroup$
      – user9077
      Dec 12 '18 at 12:04










    • $begingroup$
      Well i ruled that out since the columns and rows can get large. For $n =4$ you have to do 16 cases...for case $n$ it is general to talk about, but I found this problem in an exam paper...so something tells me our method here won't be applicable.
      $endgroup$
      – Hawk
      Dec 12 '18 at 12:25










    • $begingroup$
      Well, it can be done, I just tried it. We just need to choose $A,B$ wisely when using $g(AB)=g(BA)$.
      $endgroup$
      – user9077
      Dec 12 '18 at 12:53
















    1












    $begingroup$

    Let $E_1=begin{pmatrix}1&0\0&0end{pmatrix},E_2=begin{pmatrix}0&1\0&0end{pmatrix},E_3=begin{pmatrix}0&0\1&0end{pmatrix}$ and $E_4=begin{pmatrix}0&0\0&1end{pmatrix}$.



    If we can show that $$g(E_1)=g(E_4)=k text{ and } g(E_2)=g(E_3)=0,quad (*)$$ then
    $$gbegin{pmatrix}a&b\c&dend{pmatrix}=ag(E_1)+bg(E_2)+cg(E_3)+dg(E_4)=k(a+d)=kcdottext{tr}(A)$$
    as we want. Hence it is enough to show that $g$ satisfies $(*)$.



    Note that by the condition $g(AB)=g(BA)$ we have
    $$g(E_2)=g(E_1E_2)=g(E_2E_1)=g(0)=0$$
    and
    $$g(E_3)=g(E_4E_3)=g(E_3E_4)=g(0)=0 $$
    Now
    $(E_1+E_3)(E_1+E_2)=E_1+E_2+E_3+E_4$ and $(E_1+E_2)(E_1+E_3)=2E_1$. Then
    $$g(2E_1)=g(E_1+E_2+E_3+E_4)=g(E_1)+g(E_4)$$ which implies that
    $$g(E_1)=g(E_4)=k text{ for some } kin mathbb{R}.$$
    Hence we are done.






    share|cite|improve this answer









    $endgroup$













    • $begingroup$
      So I just checked again and apparently the original problem was stated for $M_n$, I just wrote it $2$ because that's what I wrote on my paper when solving the question and translate it incorrectly.
      $endgroup$
      – Hawk
      Dec 12 '18 at 11:50










    • $begingroup$
      Oh no. Maybe we can use the same idea?
      $endgroup$
      – user9077
      Dec 12 '18 at 12:04










    • $begingroup$
      Well i ruled that out since the columns and rows can get large. For $n =4$ you have to do 16 cases...for case $n$ it is general to talk about, but I found this problem in an exam paper...so something tells me our method here won't be applicable.
      $endgroup$
      – Hawk
      Dec 12 '18 at 12:25










    • $begingroup$
      Well, it can be done, I just tried it. We just need to choose $A,B$ wisely when using $g(AB)=g(BA)$.
      $endgroup$
      – user9077
      Dec 12 '18 at 12:53














    1












    1








    1





    $begingroup$

    Let $E_1=begin{pmatrix}1&0\0&0end{pmatrix},E_2=begin{pmatrix}0&1\0&0end{pmatrix},E_3=begin{pmatrix}0&0\1&0end{pmatrix}$ and $E_4=begin{pmatrix}0&0\0&1end{pmatrix}$.



    If we can show that $$g(E_1)=g(E_4)=k text{ and } g(E_2)=g(E_3)=0,quad (*)$$ then
    $$gbegin{pmatrix}a&b\c&dend{pmatrix}=ag(E_1)+bg(E_2)+cg(E_3)+dg(E_4)=k(a+d)=kcdottext{tr}(A)$$
    as we want. Hence it is enough to show that $g$ satisfies $(*)$.



    Note that by the condition $g(AB)=g(BA)$ we have
    $$g(E_2)=g(E_1E_2)=g(E_2E_1)=g(0)=0$$
    and
    $$g(E_3)=g(E_4E_3)=g(E_3E_4)=g(0)=0 $$
    Now
    $(E_1+E_3)(E_1+E_2)=E_1+E_2+E_3+E_4$ and $(E_1+E_2)(E_1+E_3)=2E_1$. Then
    $$g(2E_1)=g(E_1+E_2+E_3+E_4)=g(E_1)+g(E_4)$$ which implies that
    $$g(E_1)=g(E_4)=k text{ for some } kin mathbb{R}.$$
    Hence we are done.






    share|cite|improve this answer









    $endgroup$



    Let $E_1=begin{pmatrix}1&0\0&0end{pmatrix},E_2=begin{pmatrix}0&1\0&0end{pmatrix},E_3=begin{pmatrix}0&0\1&0end{pmatrix}$ and $E_4=begin{pmatrix}0&0\0&1end{pmatrix}$.



    If we can show that $$g(E_1)=g(E_4)=k text{ and } g(E_2)=g(E_3)=0,quad (*)$$ then
    $$gbegin{pmatrix}a&b\c&dend{pmatrix}=ag(E_1)+bg(E_2)+cg(E_3)+dg(E_4)=k(a+d)=kcdottext{tr}(A)$$
    as we want. Hence it is enough to show that $g$ satisfies $(*)$.



    Note that by the condition $g(AB)=g(BA)$ we have
    $$g(E_2)=g(E_1E_2)=g(E_2E_1)=g(0)=0$$
    and
    $$g(E_3)=g(E_4E_3)=g(E_3E_4)=g(0)=0 $$
    Now
    $(E_1+E_3)(E_1+E_2)=E_1+E_2+E_3+E_4$ and $(E_1+E_2)(E_1+E_3)=2E_1$. Then
    $$g(2E_1)=g(E_1+E_2+E_3+E_4)=g(E_1)+g(E_4)$$ which implies that
    $$g(E_1)=g(E_4)=k text{ for some } kin mathbb{R}.$$
    Hence we are done.







    share|cite|improve this answer












    share|cite|improve this answer



    share|cite|improve this answer










    answered Dec 12 '18 at 2:47









    user9077user9077

    1,239612




    1,239612












    • $begingroup$
      So I just checked again and apparently the original problem was stated for $M_n$, I just wrote it $2$ because that's what I wrote on my paper when solving the question and translate it incorrectly.
      $endgroup$
      – Hawk
      Dec 12 '18 at 11:50










    • $begingroup$
      Oh no. Maybe we can use the same idea?
      $endgroup$
      – user9077
      Dec 12 '18 at 12:04










    • $begingroup$
      Well i ruled that out since the columns and rows can get large. For $n =4$ you have to do 16 cases...for case $n$ it is general to talk about, but I found this problem in an exam paper...so something tells me our method here won't be applicable.
      $endgroup$
      – Hawk
      Dec 12 '18 at 12:25










    • $begingroup$
      Well, it can be done, I just tried it. We just need to choose $A,B$ wisely when using $g(AB)=g(BA)$.
      $endgroup$
      – user9077
      Dec 12 '18 at 12:53


















    • $begingroup$
      So I just checked again and apparently the original problem was stated for $M_n$, I just wrote it $2$ because that's what I wrote on my paper when solving the question and translate it incorrectly.
      $endgroup$
      – Hawk
      Dec 12 '18 at 11:50










    • $begingroup$
      Oh no. Maybe we can use the same idea?
      $endgroup$
      – user9077
      Dec 12 '18 at 12:04










    • $begingroup$
      Well i ruled that out since the columns and rows can get large. For $n =4$ you have to do 16 cases...for case $n$ it is general to talk about, but I found this problem in an exam paper...so something tells me our method here won't be applicable.
      $endgroup$
      – Hawk
      Dec 12 '18 at 12:25










    • $begingroup$
      Well, it can be done, I just tried it. We just need to choose $A,B$ wisely when using $g(AB)=g(BA)$.
      $endgroup$
      – user9077
      Dec 12 '18 at 12:53
















    $begingroup$
    So I just checked again and apparently the original problem was stated for $M_n$, I just wrote it $2$ because that's what I wrote on my paper when solving the question and translate it incorrectly.
    $endgroup$
    – Hawk
    Dec 12 '18 at 11:50




    $begingroup$
    So I just checked again and apparently the original problem was stated for $M_n$, I just wrote it $2$ because that's what I wrote on my paper when solving the question and translate it incorrectly.
    $endgroup$
    – Hawk
    Dec 12 '18 at 11:50












    $begingroup$
    Oh no. Maybe we can use the same idea?
    $endgroup$
    – user9077
    Dec 12 '18 at 12:04




    $begingroup$
    Oh no. Maybe we can use the same idea?
    $endgroup$
    – user9077
    Dec 12 '18 at 12:04












    $begingroup$
    Well i ruled that out since the columns and rows can get large. For $n =4$ you have to do 16 cases...for case $n$ it is general to talk about, but I found this problem in an exam paper...so something tells me our method here won't be applicable.
    $endgroup$
    – Hawk
    Dec 12 '18 at 12:25




    $begingroup$
    Well i ruled that out since the columns and rows can get large. For $n =4$ you have to do 16 cases...for case $n$ it is general to talk about, but I found this problem in an exam paper...so something tells me our method here won't be applicable.
    $endgroup$
    – Hawk
    Dec 12 '18 at 12:25












    $begingroup$
    Well, it can be done, I just tried it. We just need to choose $A,B$ wisely when using $g(AB)=g(BA)$.
    $endgroup$
    – user9077
    Dec 12 '18 at 12:53




    $begingroup$
    Well, it can be done, I just tried it. We just need to choose $A,B$ wisely when using $g(AB)=g(BA)$.
    $endgroup$
    – user9077
    Dec 12 '18 at 12:53











    0












    $begingroup$

    We can use the same idea if $g:M_n(mathbb{R})tomathbb{R}$. The idea is that if $B$ is a basis of $M_n$, also $g$ and $text{tr}$ agree on $B$ up to some constant multiple $alpha$, then $$g(A)=alphacdot text{tr}(A) $$ for all $Ain M_n$.



    Let $E_{i,j}$ be the matrix where all the entries are zero except it is 1 at the position $(i,j)$. Let $T_k=text{diag}(underbrace{1,1,ldots,1}_k,0,ldots,0)$. Notice that for $ineq j$ we have $E_{i,j}T_j=E_{i,j}$ while $T_jE_{i,j}=0$. Therefore for all $ineq j$ we have $g(E_{i,j})=0$.



    Now let $J_k$ be a $ktimes n$ matrix where all the entries are 1. Define an $ntimes n$ matrix $S_k$ as a block matrix $$S_k:=begin{pmatrix}J_k\0end{pmatrix}$$.



    Note that since $g(E_{i,j})=0$ for $ineq j$, we have $g(S_k)=g(T_k)$ for all $k$.



    Now
    begin{align*}
    S_kS_1&=S_k \ S_1S_k&=kS_1
    end{align*}



    Hence $g(T_k)=g(S_k)=g(kS_1)=kg(S_1)=kg(T_1)$. Notice that the trace map also satisfies $text{tr}(T_k)=k$. By taking $alpha=g(T_1)$ then $g$ and $alpha cdot text{tr}$ are two linear maps that agree on $B={E_{i,j}mid ineq j}cup {T_kmid k=1,..,n}$ which is a basis of $M_n$. Hence $g=alphacdot text{tr}.$






    share|cite|improve this answer









    $endgroup$


















      0












      $begingroup$

      We can use the same idea if $g:M_n(mathbb{R})tomathbb{R}$. The idea is that if $B$ is a basis of $M_n$, also $g$ and $text{tr}$ agree on $B$ up to some constant multiple $alpha$, then $$g(A)=alphacdot text{tr}(A) $$ for all $Ain M_n$.



      Let $E_{i,j}$ be the matrix where all the entries are zero except it is 1 at the position $(i,j)$. Let $T_k=text{diag}(underbrace{1,1,ldots,1}_k,0,ldots,0)$. Notice that for $ineq j$ we have $E_{i,j}T_j=E_{i,j}$ while $T_jE_{i,j}=0$. Therefore for all $ineq j$ we have $g(E_{i,j})=0$.



      Now let $J_k$ be a $ktimes n$ matrix where all the entries are 1. Define an $ntimes n$ matrix $S_k$ as a block matrix $$S_k:=begin{pmatrix}J_k\0end{pmatrix}$$.



      Note that since $g(E_{i,j})=0$ for $ineq j$, we have $g(S_k)=g(T_k)$ for all $k$.



      Now
      begin{align*}
      S_kS_1&=S_k \ S_1S_k&=kS_1
      end{align*}



      Hence $g(T_k)=g(S_k)=g(kS_1)=kg(S_1)=kg(T_1)$. Notice that the trace map also satisfies $text{tr}(T_k)=k$. By taking $alpha=g(T_1)$ then $g$ and $alpha cdot text{tr}$ are two linear maps that agree on $B={E_{i,j}mid ineq j}cup {T_kmid k=1,..,n}$ which is a basis of $M_n$. Hence $g=alphacdot text{tr}.$






      share|cite|improve this answer









      $endgroup$
















        0












        0








        0





        $begingroup$

        We can use the same idea if $g:M_n(mathbb{R})tomathbb{R}$. The idea is that if $B$ is a basis of $M_n$, also $g$ and $text{tr}$ agree on $B$ up to some constant multiple $alpha$, then $$g(A)=alphacdot text{tr}(A) $$ for all $Ain M_n$.



        Let $E_{i,j}$ be the matrix where all the entries are zero except it is 1 at the position $(i,j)$. Let $T_k=text{diag}(underbrace{1,1,ldots,1}_k,0,ldots,0)$. Notice that for $ineq j$ we have $E_{i,j}T_j=E_{i,j}$ while $T_jE_{i,j}=0$. Therefore for all $ineq j$ we have $g(E_{i,j})=0$.



        Now let $J_k$ be a $ktimes n$ matrix where all the entries are 1. Define an $ntimes n$ matrix $S_k$ as a block matrix $$S_k:=begin{pmatrix}J_k\0end{pmatrix}$$.



        Note that since $g(E_{i,j})=0$ for $ineq j$, we have $g(S_k)=g(T_k)$ for all $k$.



        Now
        begin{align*}
        S_kS_1&=S_k \ S_1S_k&=kS_1
        end{align*}



        Hence $g(T_k)=g(S_k)=g(kS_1)=kg(S_1)=kg(T_1)$. Notice that the trace map also satisfies $text{tr}(T_k)=k$. By taking $alpha=g(T_1)$ then $g$ and $alpha cdot text{tr}$ are two linear maps that agree on $B={E_{i,j}mid ineq j}cup {T_kmid k=1,..,n}$ which is a basis of $M_n$. Hence $g=alphacdot text{tr}.$






        share|cite|improve this answer









        $endgroup$



        We can use the same idea if $g:M_n(mathbb{R})tomathbb{R}$. The idea is that if $B$ is a basis of $M_n$, also $g$ and $text{tr}$ agree on $B$ up to some constant multiple $alpha$, then $$g(A)=alphacdot text{tr}(A) $$ for all $Ain M_n$.



        Let $E_{i,j}$ be the matrix where all the entries are zero except it is 1 at the position $(i,j)$. Let $T_k=text{diag}(underbrace{1,1,ldots,1}_k,0,ldots,0)$. Notice that for $ineq j$ we have $E_{i,j}T_j=E_{i,j}$ while $T_jE_{i,j}=0$. Therefore for all $ineq j$ we have $g(E_{i,j})=0$.



        Now let $J_k$ be a $ktimes n$ matrix where all the entries are 1. Define an $ntimes n$ matrix $S_k$ as a block matrix $$S_k:=begin{pmatrix}J_k\0end{pmatrix}$$.



        Note that since $g(E_{i,j})=0$ for $ineq j$, we have $g(S_k)=g(T_k)$ for all $k$.



        Now
        begin{align*}
        S_kS_1&=S_k \ S_1S_k&=kS_1
        end{align*}



        Hence $g(T_k)=g(S_k)=g(kS_1)=kg(S_1)=kg(T_1)$. Notice that the trace map also satisfies $text{tr}(T_k)=k$. By taking $alpha=g(T_1)$ then $g$ and $alpha cdot text{tr}$ are two linear maps that agree on $B={E_{i,j}mid ineq j}cup {T_kmid k=1,..,n}$ which is a basis of $M_n$. Hence $g=alphacdot text{tr}.$







        share|cite|improve this answer












        share|cite|improve this answer



        share|cite|improve this answer










        answered Dec 12 '18 at 13:28









        user9077user9077

        1,239612




        1,239612






























            draft saved

            draft discarded




















































            Thanks for contributing an answer to Mathematics Stack Exchange!


            • Please be sure to answer the question. Provide details and share your research!

            But avoid



            • Asking for help, clarification, or responding to other answers.

            • Making statements based on opinion; back them up with references or personal experience.


            Use MathJax to format equations. MathJax reference.


            To learn more, see our tips on writing great answers.




            draft saved


            draft discarded














            StackExchange.ready(
            function () {
            StackExchange.openid.initPostLogin('.new-post-login', 'https%3a%2f%2fmath.stackexchange.com%2fquestions%2f3036061%2fshowing-two-linear-functionals-are-the-same-up-to-constant-ga-qtra%23new-answer', 'question_page');
            }
            );

            Post as a guest















            Required, but never shown





















































            Required, but never shown














            Required, but never shown












            Required, but never shown







            Required, but never shown

































            Required, but never shown














            Required, but never shown












            Required, but never shown







            Required, but never shown







            Popular posts from this blog

            How do I know what Microsoft account the skydrive app is syncing to?

            When does type information flow backwards in C++?

            Grease: Live!